- PowerScore Staff
- Posts: 5972
- Joined: Mar 25, 2011
- Mon Mar 12, 2012 3:29 pm
#85364
Complete Question Explanation
(The complete setup for this game can be found here: lsat/viewtopic.php?f=315&t=9170)
The correct answer choice is (C)
The question stem indicates that K views the site on day 4, and that a JM block is in effect. If K views the site on day 4, then from the fourth rule L views the site on day 5:
Because L views the site on day 5, from the first rule F views the site on day 3:
At this point, the JM block is still unplaced, as is the HG block from the fifth rule. However, the JM block cannot be placed on days 1 and 2 because that would violate the third rule. Thus, the JM block must be placed on days 6 and 7, leaving the HG block on days 1 and 2:
Accordingly, answer choice (C) is correct.
(The complete setup for this game can be found here: lsat/viewtopic.php?f=315&t=9170)
The correct answer choice is (C)
The question stem indicates that K views the site on day 4, and that a JM block is in effect. If K views the site on day 4, then from the fourth rule L views the site on day 5:
Because L views the site on day 5, from the first rule F views the site on day 3:
At this point, the JM block is still unplaced, as is the HG block from the fifth rule. However, the JM block cannot be placed on days 1 and 2 because that would violate the third rule. Thus, the JM block must be placed on days 6 and 7, leaving the HG block on days 1 and 2:
Accordingly, answer choice (C) is correct.
You do not have the required permissions to view the files attached to this post.
Dave Killoran
PowerScore Test Preparation
Follow me on X/Twitter at http://twitter.com/DaveKilloran
My LSAT Articles: http://blog.powerscore.com/lsat/author/dave-killoran
PowerScore Podcast: http://www.powerscore.com/lsat/podcast/
PowerScore Test Preparation
Follow me on X/Twitter at http://twitter.com/DaveKilloran
My LSAT Articles: http://blog.powerscore.com/lsat/author/dave-killoran
PowerScore Podcast: http://www.powerscore.com/lsat/podcast/